ATI MED SURG PROCTORED FINAL EXAM 2019 TEST BANK CONTAINS 318 QUESTIONS AND DETAILED ANSWERS WITH RATIONALES|AGRADE

ATI ADVANCED MED SURG PROCTORED 2019 EXAM GUIDE

1. A nurse in a burn treatment center is caring for a client who is admitted with

severe burns to both lower extremities and is pending an escharotomy. The client’s

spouse asks the nurse what the procedure entails. Which of the following

nursing statements is appropriate?

a. “large incisions will be made in the eschar to improve circulation”

b. “ I can call the doctor back here if you want me to”

c. “a piece of skin will be removed and grafted over the burned area”

d. “dead tissue will be surgically removed”

2. A nurse is monitoring the fluid replacement of a client who has sustained burns.

Which of the following fluids is used in the first 24 hours following a burn injury?

a. 5% dextrose in water

b. 5% dextrose in normal saline

c. normal saline

d. lactated ringers

3. A nurseiscaringforaclientwhohasfull-thickness burnsall over75%ofhis

body.Whichofthefollowingmethodsisappropriatetoaccuratelymonitorthe

cardiovascular system?

a. auscultate cuff blood pressure

b. palpate pulse pressure

c. obtain a central venous pressure

d. monitor the pulmonary artery pressure

4. A nurse is assessing the depth and extent of a client who has severe burns to the

face, neck, and upper extremities. Which of the following factors is the first priority

when assessing the severity of the burn?

a. Age of the client

b. Associated medical history

c. Location of the burn

d. Cause of the burn

5. A client arrives at the emergency dept following an explosion at the chemical

plant. He has deep partial and full-thickness chemical burns over more than 25 %

of his body surface area. What is the nurse’s priority intervention?

a. Initiate fluid resuscitation

b. Medication for pain

c. Administer antibiotics

d. Maintain a patent airway

6. A nurseis caringfor aclientwho camethe emergencydeptreportingchestpain.

Theprovidersuspectsamyocardialinfarction.Whilewaitingforthelaboratoryto

report the client’s troponin levels, the client asks what this blood test will show.

The nurse should explain that troponin is

a. An enzyme that indicates damage to brain, heart, and skeletal muscle

tissues

b. A protein whose levels reflect the risk for coronary artery disease

c. A heart muscle protein that appears in the bloodstream when there is

damage to the heart

d. A protein that helps transport oxygen throughout the body

7. A nurse is assessing a client who has disseminated intravascular coagulation

(DIC). Which of the following should the nurse expect in the findings?

a. Excessive thrombosis and bleeding

b. Progressive increase in platelet production

c. Immediate sodium and fluid retention

d. Increased clotting factors

8. A nurse is about to administer warfarin (Coumadin) to a client who has atrial

fibrillation. When the client asks what his medication will do, which of the

following is an appropriate nursing response?

a. It helps convert atrial fibrillation to sinus rhythm

b. Is dissolves clots in the bloodstream

c. It slows the response of the ventricles to the fast atrial impulses

d. It prevents strokes in clients who have atrial fibrillation

9. A nurse in a cardiac care unit is caring for a client with acute heart failure. Which

of the following findings should the nurse expect?

a. Decreased brain natriuretic peptide (BNP)

b. Elevated central venous pressure (CVP)

c. Decreased pulmonary pressure

d. Increases urinary output

10. A client comes into the ED reporting nausea and vomiting that worsens when

lying down and without relief from antacids. The provider suspects acute

pancreatitis. Which of the following lab test results should the nurse expect to see if

the client has acute pancreatitis?

a. Decreased WBC

b. Increased serum amylase

c. Decreased serum lipase

d. Increased serum calcium

11. A nurse in the ICU is caring for a client who has acute respiratory distress

syndrome (ARDS) and is receiving mechanical via an endotracheal tube. The

provider plans to extubate her within the next 24 hour. Which of the following is

an important criterion for extubating the client?

a. Ability to cough effectively

b. Adequate tidal volume without manually assisted breaths

c. No indication of infection

d. No need for supplemental oxygen

12. A nurse is caring for a client following a CT scan with dye who suffered from an

anaphylactic reaction. Which of the following conditions requires a priority nursing

response?

a. urticaria

b. stridor

c. tachypnea

d. angioedema

13. A nurse is caring for a female client who came in to the ED reporting SOB and

pain in the lung area. Her heart rate is 110/min, resp. rate 40/min, and blood

pressure 140/80 m8juiimHg. Her arterial blood gases are: pH 7.5, PaCO2 29 mmHg,

PaO2 60 mm Hg, HCO3 20 mEq/L, and SaO2 86%. Which of the following is the

priority intervention?

a. Prepare for mechanical ventilation

b. Administer oxygen via face mask

c. Prepare to administer a sedative

d. Monitor for pulmonary embolism

14. A nurse is monitoring a client who has just had a thoracentesis to remove pleural

fluid. Which of the following clinical manifestations indicate a complication that

requires notifying the provider immediately?

a. Serosanguineous drainage from the puncture site

b. Discomfort at the puncture site

c. Increased heart rate

d. Decreased temperature

15. A group of college students was attending a weekend football rally when one of

the students stumbled and fell into the bonfire. Although several friends quickly

intervened, the client sustained partial-thickness burns to both lower legs, chest,

and both forearms. Which of the following is priority nursing action when the

client is brought to the ED?

a. cover the burned area with sterile gauze

b. inspect mouth for signs of inhalation

c. administer intravenous pain medication

d. draw blood for a CBC

16. A triage nurse in an emergency dept is caring for a client who has gunshot

wound to the right side of chest. The nurse notices thick dressing on the chest and

sucking noise coming from the wound. The client has a blood pressure of 100/60

mm Hg, a weak pulse rate of 118/min, and a respiratory rate of 40/min. Which of

the following actions should the nurse take initially?

a. Raise the foot of the bed to a 90 degree angle

b. Remove the dressing to inspect the wound

c. Prepare to insert a central line

d. Administer oxygen via nasal cannula

17. A nurse is suctioning the endotracheal tube of a client who is on a ventilator. The

client’s heart rate increases from 86/min to 110/min and becomes irregular. The

nurse should know that the client requires which of the following?

a. A cardiology consult

b. Less frequent suctioning

c. An antidysrhythmic medication

d. Pre-oxygenation prior to suctioning

18. The nurse is caring for a client who is receiving a blood transfusion. The

transfusion started 30 minutes ago at a rate of 100 mL/hr. The client begins to

complain of low back pain and headache and is increasingly restless. What is the

first nursing action?

a. Stop the transfusion, disconnect the blood tubing, and begin a

primary infusion of normal saline solution

b. Slow the infusion and evaluate the vital signs and the client’s history of

tranfusion reactions

c. Slow the infusion of blood and begin infusion of normal saline solution

from the Y connector.

d. Recheck the unit of blood for correct identification numbers and

crossmatch information

19. A client with a diagnosis of disseminated intravascular coagulation (DIC) has the

following assessment findings: blood pressure of 76/56, temperature 102.6

degrees, resp. 24 breath/min., with complaints of severe neck and back pain.

Which nursing action should the nurse implement first?

a. Administer acetaminophen (Tylenol) PO.

b. Administer ibuprofen (Motrin) PO.

c. Draw coagulation study blood work in the AM

d. Give morphine sulfate IV

20. The nurse administering albuterol (Proventil) via a metered-dose inhaler (MDI)

to a client who has a history of coronary artery disease is now in congestive heart

failure. What side effects will be particularly important to observe for when the client

takes the medication?

a. Tremors and central nervous system stimulation

b. Tachycardia and chest discomfort

c. Development of oral candidiasis

d. An increase in blood pressure

21. The nurse is assessing a client who is on a ventilator and has an endotracheal

tube in place. What data confirms that the tube has migrated too far into the

trachea?

a. Decreased breath sounds are heard over the left side of the chest

b. Increased rhonchi are present at the lung bases bilaterally

c. Ventilator pressure alarm continues to sound

d. Client is able to speak and coughs excessively

22. What is the desired action of dopamine (Intropin) when administered in the

treatment of shock?

a. It increases myocardial contractility

b. It is associated with fewer severe allergic reactions

c. It causes rapid vasodilation of the vascular bed

d. It supports renal perfusion by dilation of the renal arteries

23. The nurse is monitoring an IV infusion of sodium nitroprusside (Nirpride). Fifteen

minutes after the infusion is started, the client’s BP goes from 190/120 mm Hg to

120/90 mm Hg. What is the priority nursing action?

a. Recheck the BP and call the doctor

b. Decrease the infusion rate and recheck the blood pressure in 5

minutes

c. Stop the medication and keep the IV open with D5W.

d. Assess the client’s tolerance of the current level of BP

24. Norepinephrine (Levophed) has been ordered for a client in hypovolemic shock.

Before administering the drug, the nurse should make sure that the client has:

a. A heart rate of less than 120 beats/min

b. Urine output of at least 30 mL/hr.

c. Received adequate anticoagulation

d. Been receiving adequate IV fluid replacement

25. The client returns to his room after a thoracotomy. What will the nursing

assessment reveal if hypovolemia from excessive blood loss is present?

a. CVP of 3 cm H20 and urine output of 20 mL/hr

b. Jugular vein distention with the head elevated 45 degrees

c. Chest tube drainage of 50 mL/hr in the first 2 hours

d. Persistent increased BP and increased pulse pressure

26. The nurse is performing an assessment and finds the client has cold, clammy

skin, pulse of 130 beats/min and weak, blood pressure of 84/56 mm Hg, and

urinary of 20 mL for the past hour. The nurse would interpret these findings as

suggestive of which pathophysiology?

a. Reduction of circulation to the coronary arteries, this increasing the

preload

b. Decreased glomeruli filtration rate, resulting in volume overload

c. Stimulation of the sympathetic nervous system, causing severe

vasoconstriction

d. Decrease in the cardiac output and inadequate tissue perfusion

27. The nurse applies a Nitro-Dur patch on a client who has undergone cardiac

surgery. What nursing observation indicates that a Nitro-Dur patch is achieving

the desired effect?

a. Chest pain is completely relieved

b. Client performs activities of daily living without chest pain

c. Pain is controlled with frequent changes of patch

d. Client tolerates increased activity without pain

28) The V/S of a client with Cardiac disease are as follows: BP 102/76 mm/hg, Pulse

52, RR 16. Atropine is administered IV push. What nursing assessment indicates a

therapeutic response to the medication?

A. Pulse rate has increased to 70 beats/min

B. systolic BP has increased by 20

C. pupils are dilated

D. oral secretions have decreased

29) An older adult client comes into ER stating that he has no appetite, is nauseated,

his heart feels funny and has noticed a haziness in his vision. The client states that he

has been taking an antihypertensive drug and digitalis for more than a year.

Based on the presenting symptoms, what would be the priority nursing action?

A. Obtain an order for an EKG and serum potassium and digitalis levels

B. Perform a neurological assessment to determine whether he has one side

weakness.

C. Assess lungs for decreased breath sounds and/or adventitious breath

sounds.

d. Obtain an order for an EKG

30) the nurse is administering alteplase to a client who has been diagnosed with

acute coronary syndrome. What are important nursing implications for this

medication?

A. Monitor the ECG for dysrthymias

B. Place the client on bleeding precautions

C. monitor urine output hourly

D. Monitor for activity tolerance

31) The nurse is caring for a client who underwent cardiac catheterization 1 hour

ago. What is an important nursing measure at this time?

A. Measure urinary output hourly and maintain continuous cardiac monitoring

B. Encourage client to perform slow pressure exercise of the affected side to

promote circulation.

C. Maintain pressure over catheter insertion site and determine distal

circulation status.

D. Evaluate apical pulse and determine presence of pulse deficit.

32. The nurse in a cardiac stepdown unit has received a hand-off shift report for

these clients. Which client should be assess first?

A. a client who has just returned from a coronary artierogram with placement

of an intracoronary stent.

B. A client who is in heart failure and has gained 2 pnds in the last 24 hours.

C. a client with endocarditis who has temperature elevation of 100F and P 100

beats/min

D. A client who was cardioverted from atrial fib 24 hours ago and has had 3 atrial

premature

33) What ECG changes would reflect myocardial ischemia in a client who has been

admitted for observation after experiencing an episode of chest pain?

A. Prolonged PR interval

B. Wide QRS complex

C. ST- Segment elevation or depression

D. Tall, peak T-waves

34) A new employee at a facility needs a hepatitis vaccine. Which statement reflects

accurate understanding of the immunization?

A. I need to get 6 shots of hep C

B. Once I receive the Hep vaccine I will always been immune

C. I will receive 3 injections over a period of months, which should protect me

from hep B

D. Hep vaccine is an oral vaccine with live attenuated Virus

35) While talking with a client with a diagnosis of end stage liver disease. The nurse

notices the client is unable to stay awake and seems to fall asleep in the middle of a

sentence. The nurse recognizes these symptoms to be indicative of what condition?

A. Hyperglycemia

B. Increased Bile production

C. Increased blood ammonia levels

D. Hypocalcaemia

36) The nurse is caring for a client with chronic hep B. What will the teaching plan

for this client include?

A. use a condom for sexual intercourse

B. Report any clay- colored stools.

C. Eat a high protein diet

D. Perform daily urine bilirubin checks

37. A patient with massive trauma and possible spinal cord injury is admitted to the

emergency department (ED). Which finding by the nurse will help confirm a diagnosis

of neurogenic shock?

a. cool clammy skin

b. inspiratory crackles

c. apical heart rate of 48 beats/min

d. temperature 101.2* F

38. Apatientwithsepticshockhasaurineoutputof20mL/hrforthepast3hours.

Thepulserateis120andthecentralvenousandpulmonaryarterywedgepressure

are 4. Which of these orders by the health care provider will the nurse question?

a. Give furosemide (Lasix) 40 mg IV

b. increase normal saline infusion to 150 mL/hr

c. Administer hydrocortisone (SoluCortef) 100 mg IV

d. Prepare to give drotrecogin alpha (Xigris) 24 mcg/kg/hr

39. After receiving 1000 mL of normal saline, the central venous pressure for a

patient who has septic shock is 10 mm Hg, but the blood pressure is still 82/40 mm

Hg. The nurse will anticipate the administration of which of the following?

a. Nitroglycerin (Tridil)

b. Sodium nitroprusside (Nipride)

c. Drotrecogin alpha (Xigris)

d. Norepinephrine (Levophed)

40. Which of these findings is the best indicators that the fluid resuscitation for a

patient with hypovolemic shock has been successful?

a. hemoglobin is within normal limits

b. Urine output is 60 mL over the last hour

c. Pulmonary artery wedge pressure (PAWP) is 10 mmHg

d. Mean arterial pressure (MAP) is 55 mm Hg

41. Which interventions will the nurse include in the plan of the care for a patient

who has cardiogenic shock?

a. Avoid elevating head of bed

b. Check temperature every 2 hours

c. Monitor breath sounds frequently

d. Assess skin for flushing and itching

42. Which assessment is most important for the nurse to make in order to evaluate

whether treatment of a patient with anaphylactic shock has been effective?

a. Pulse rate

b. Orientation

c. Blood pressure

d. Oxygen saturation

43. When caring for the patient who has septic shock, which assessment finding is

most important for the nurse to report to the health care provider? (TB ch.67 Q.17)

a. BP 92/56 mm Hg

b. Skin cool and clammy

c. apical pulse 118 beats/min

d. Arterial oxygen saturation 91%

44. During change-of-shift report, the nurse learns that a patient has been admitted

with dehydration and hypotension after having vomiting and diarrhea for 3 days.

Which findings is most important for the nurse to report to the HCP?

a. Decreased bowel sounds

b. Apical pulse 110 beats/min

c. Pale, cool, and dry extremities

d. New onset of confusion and agitation

45. A patient is admitted to the burn unit with burns the upper body and head after a

garage fire. Initially, wheezes are heard, but an hour later, the lung sounds are

decreased ad no wheezes are audible. What is the best action for the nurse to take?

a. encourage the patient to cough and auscultate the lungs again

b. Notify the HCP and prepare for endotracheal intubation

c. Document the results and continue to monitor the patient’s resp. rate

d. Reposition pt in high-Fowler’s position and reassess breath sounds

46. During the emergent phase of burn care, which nursing action will be most

useful in determining whether the patient is receiving adequate fluid infusion?

a. Check skin turgor

b. Monitor daily weight

c. Assess mucous membranes

d. Measures hourly urine output

47. After receiving change-of-shift report, which of these patients should the nurse

assess first?

a. A patient with smoke inhalation who has wheezes and altered mental

status

b. A patient with full-thickness leg burns who has a dressing change scheduled

c. A patient with abdominal burns who is complaining of level 8 (0 to 10 scale)

pain.

d. A patient with 40% total body surface area (TBSA) burns who is receiving IV

fluids at 500 mL/hr

48. The RN observes all of the following actions begin taken by a staff nurse who has

floated to the unit. Which action requires that the RN intervene?

a. The nurse uses latex gloves when applying antibacterial cream to a burn

wound

b. The float nurse obtains burn cultures when the patient has a temp of 101* F

c. The float nurse administers PRN fentanyl (Sublimaze) IV to a pt 5 minutes

before a dressing change

d. The float nurse calls the health care provider for an insulin order when a

nondiabetic pt has an elevated serum glucose

49) A client with cervical neck fracture is admitted to the intensive care unit. Which

findings would the nurse recognize as indicative of spinal shock?

A. Spastically, neuromuscular irritability, hyperreflexia

B. Flaccidity and lack of sensation below the level of spinal cord lesion.

C. Automatic dysreflexia with neurogenic bladder symptoms

D. Muscular spasticity and loss of motor reflexes in all parts of the body below the

level of spinal cord lesion.

****know T2-T3: paraplegic ***************

50) A client with T6 spinal cord injury is being discharged. The PT is concerned

about autonomic dysreflexia. S/S include the following:

A. Dialited pupils

B. Sudden vomiting and diarrhea

C. drop in BP and pulse

D. Diaphoresis above the level of the lesion

51) A woman has been recently diagnosed with systemic lupus and shares with the

nurse, I want to get pregnant, but I don’t know how I will tolerate pregnancy because

I have lupus. Which response is best?

A. Most women find that they feel better when they are pregnant

B. How long have you been in remission?

C. Women with lupus frequently have slightly longer gestation

D. Its best to become pregnant within the first 6 months of diagnosis

52. The nurse is assessing the patency of an arteriovenous fistula and suspects

clotting in the fistula if which finding are noted? Select all that apply

A. presence of a thrill on palpation over the fistula

*B. Absence of a bruit on auscultation over the fistula

C. Presence of a pulse in the extremity below the fistula

*D. Complaints of tingling or discomfort in the extremity

E. Warm hand and fingers in the extremity in which the fistula is located.

53. Epoetin alfa (Epogen) is prescribed for a client diagnosed with chronic renal

failure. The client asks the nurse about the purpose of the medication. The

appropriate response would be which of the following?

A. It is used to lower your blood pressure

*B. It is used to treat anemia

C. It will help to increase the potassium levels in your body

D. It is an anticonvulsant medication given to all clients after dialysis to prevent

seizure activity.

54. A client with and ECG reading showing sinus bradycardia has a blood pressure of

47/28 mmhg. Which drugs does the nurse expect the physician to order for this client?

A. Lidocaine (Xylocaine)

*B. Atropine sulfate

C. Isoproterenol hydrochloride (Isuprel)

D. Epinephrine

55. Chemical cardioversion is prescribed for the client with atrial fibrillation. The

nurse who is assisting in preparing the client would expect that which medication

specific for chemical cardioversion will be needed?

A. Nitroglycerin

B Nifedipine (Procardia)

C Lidocaine (Xylocaine)

*D. Amiodarone (Cordarone)

56. A nurseassessesacomatose,head-injuredclientandfindsflexionofthearms,

wrists,andfingersandadductionoftheupperextremities.Whichofthefollowing

describes these findings?

A. Stroke

B. Epileptic Seizure

*C. Decorticate posturing

D. Decerebrate posturing

57. The client diagnosed with ARDS is transferred to the intensive care department

and placed on a ventilator. Which intervention should the nurse implement first?

A Confirm that the ventilator settings are correct

B Verify that the ventilator alarms are functioning properly

*C. Assess the respiratory status and pulse oximeter reading.

D Monitor the clients arterial blood gas results.

58. The low-pressure alarm sounds on a ventilator. A nurse assesses the client and

then attempts to determine the cause of the alarm. The nurse is unsuccessful in

determining the cause of the alarm and takes what initial action?

A. Administer oxygen

B. Checks the client’s vital signs

*C Ventilates the client manually

D. Starts cardiopulmonary resuscitation

59. The client is admitted to the ED with chest trauma. Which signs/symptoms

would the nurse expect to assess that supports the diagnosis of pneumothorax?

A. Bronchovesicular lung sounds and friction rub

*B Absent breath sounds and tachypnea

C Nasal flaring and lung consolidation

D Symmetrical chest expansion and bradypnea.

60. A nurse is planning care for a client with a chest tube attached to a Pleur-Evac

drainage system. The nurse includes which interventions in the plan? Select all that

apply

A. Clamping the chest tube intermittently

*B. Changing the client’s position frequently

*C. Maintaining the collection chamber below the client’s waist

*D. Adding water to the suction control chamber as it evaporates.

*E Taping the connection between the chest tube and the drainage

system.

61. A client has a total serum calcium level of 7.5 mg/dl. Which clinical manifestations

would the nurse expect to note on assessment of the client? Select all

A Constipation

*B Muscle twitches

C Hypoactive bowel sounds

*D Hyperactive deep tendon reflexes

*E Positive Trousseau’s sign and positive Chvostek’s sign

*F. Prolong ST interval and QT interval on ECG

62. The client diagnosed with rule-out myocardial infarction is experiencing chest pain

while walking to the bathroom. Which action should the nurse implement first?

A. Administer sublingual nitroglycerin.

B Obtain a STAT electrocardiogram

*C Have the client sit down immediately

D Assess the clien’ts vital signs.

63. The nurse is caring for a client diagnosed with ARDS who is on a ventilator.

Which interventions should the nurse implement. Select all

*A Assess the client’s level of consciousness

*B Monitor clients urine output

*C Perform passive range of motion exercise

*D maintain intravenous fluids as ordered

E Place the client with the HOB flat

64. The nurse is assessing a client experiencing motor loss as a result of a left sided

cerebrovascular accident (CVA). Which clinical manifestations would the nurse

document?

The most common motor dysfunction of a CVA is paralysis of one side of

the body, hemiplegia; in this case with a left-sided CVA, paralysis would affect the

right side. Ataxia is an impaired ability to coordinate movement.

65. When teaching a client about the expected outcomes after intravenous

administration of furosemide, the nurse would include which outcome?

A. Increased blood pressure

*B Increased urine output

C Decreased pain

D Decreased PVCs

66. A client arrives at the emergency department with deep partial thickness and

burns over 15% of his body. At admission his vital signs are blood pressure 100/50

mm Hg, heart rate 130 beats/minute and respiratory rate 20 breaths/minute. Which

nursing intervention are appropriate for this client? Select all that apply

A. Starting an IV infusion of lactated Ringers solution

B. Administering 6mg of morphine IV

C. Administering tetanus prophylaxis as ordered

67. If dietary trays are usually brought to the nursing unit at 8:00am the nurse should

plan to administer intermediate- acting insulin (Humlin N) 40 units SQ to the client

between?

ANSWER- 630am and 700 am

68. What ECG changes would reflect myocardial ischemia in a client who has been

admitted for observation after experiencing an episode of chest pain?

ANSWER- ST segment elevation or depression

69. The client with acute renal failure has a serum potassium level of 6.0 mEq/L. The

nurse would plan which of the following as a priority action?

ANSWER- Place the client on a cardiac monitor

67. The nurse is caring for a client who underwent cardiac catherization 1 hour ago.

What is an important nursing measures at this time?

ANSWER- MAINTAIN PRESSURE OVER CATHETER INSERTION

SITE AND DETERMINE DISTAL CIRCULATION STATUS.

68. A nurse is suctioning the endotracheal tube of a client who is on a ventilator. The

client’s heart rate increases ??? to 110min and becomes irregular. The nurse should

know that the client requires?

ANSWER- OXYGENATE PRIOR TO SUCTIONING

69. A client comes into the ER with complains of midsternal chest pain radiating to the

neck and left arm which is unrelieved by sublingual nitroglygen. An electrocardiogram

(ECG) is obtained. What observation on the ECG or on the cardiac monitor would

indicate to the nurse the need to immediately notify the physician?

A. PR impulse 0.20 sec

B. Tachycardia rate of 125 beat of premature

C. premature ventricle beat

D. An ST segment elevation from the isoelectric baseline.

70. A client begins complains of chills and discomfort after about 50ml of blood has

packed red blood cells. The best nursing action at this time is to

A. Discontinue the transfusion and move the IV and restart IV transfusion at another

site.

B. compare the VS now and what they were before the transfusion begin

C. STOP THE TRANSFUSION AND MAINTAIN A PATENT LINE WITH NORMAL

SALINE solution and new tubing

D. slow down the transfusion blood and dilute with normal saline solution

71. The vital signs of a client with cardiac disease are as follows blood pressure of

103/78 mm Hg, heart rate ??? beats/min, and respiratory rate of 16 breaths/min.

Atropine (atropine???? Administered IV push. What nursing assessment indicates a

therapeutic response to the medication?

ANSWER- Pulse rate has increased to 70 beats/min

72. Order rocephen 1g over 30minutes Q6H. Supply 1g/100mL. How many mL per

hour will the nurse infuse? Round the nearest whole number.

ANSWER- 200 ml/hr

73. The nurse is caring for client who is 1 day postoperative following an open

thoracotomy. The client is receiving oxygen mist at 40 percent. The 02 saturation

measured by pulse oximeter was 83 ABG results are pH 7.31, PACO2 93mmHg,

HCO3 25 meq/L. Which of the following is an appropriate action by the nurse?

ANSWER- POSITION CLIENT IN HIGH- FOWLERS AND ENCOURAGE

USE OF INCENTIVE SPIROMETER AND COUGHING.

74. The diabetic educator is teaching a class on Diabetes Type 1 and is discussing

sick day rules. Which interventions should the diabetes counselor include in the

teaching? Select all that apply

ANSWER-

A) Take diabetic medication even if unable to eat the client’s normal

diet. B) If unable to eat, drink liquids equal to the client’s normal diet.

D) Test the blood glucose levels and test the urine ketones once a day

and keep a record.

75. The nurse is monitoring a client receiving pertional dialysis notes that the client’s

outflow is less than inflow. What action should the nurse take? Select all that apply?

ANSWER- (SELECT ALL ANSWERS EXPCEPT – CONTACT THE HEALTH CARE

PROVIDER & INCREASE THE FLOW- DO NOT SELECT THOSE TWO ANSWERS)

76. The nurse is obtaining a health history from a client who is visiting the clinic with

complaints of a severe headache. The client provides the following data to the nurse

based on a review of systems. The nurse identifies the following as a modifiable risk

for stroke? Select all the apply.

A. SMOKING

B. ALCOHOL CONSUMPTION

C. DECREASED PHYSICAL ACTIVITY

D. OBESITY

77. The nurse is caring for a client diagnosed with ARDS who is on a ventilator.

Which intervention should the nurse implement? Select all that apply

A. Assess the client’s level of consciousness.

B. Monitor the client’s urine output

C. Perform passive range of motion (ROM) exercise.

D. Maintain intravenous fluids as ordered.

78. The nurse is performing an assessment on a client who has returned from dialysis

unit following hemodialysis. The client is complaining of headache, nausea,

and is extremely restless. Which of the following ?? the most appropriate nursing

action?

ANSWER- NOTIFY THE PHYSICANS

79. The nurse determines that a client with diabetes- mellitus is experiencing fat

breakdown for conversion to glucose if the client has elevated levels of which

substance in the urine?

ANSWER- ketones

80. The client is admitted the ED with chest trauma. Which signs and symptoms

would the nurse expect to assess that supports the diagnosis of pneumothorax?

ANSWER- ABSENT BREATHS SOUNDS TACHYPNEA

81. The nurse is caring for hospitalized clients. Which of the following clients is at

greatest risk for fluid volume deficit?

ANSWER- THE CLIENT WHO HAS JUST BEEN ADMITTED HAS SEVERE

DIARRHEA AND IS febrile.

82. PT & INR for Coumadin, INR 2.8: continue medication

83. Hot spot: apical pulse

84. Hot spot: T wave

85. Hot spot: P wave

86. P wave: atrial depolarization

87. Assessing response in an unconscious patient: nail bed pressure (peripheral)

88. HbA1c considerations for about 3 months of glucose monitoring

a. less than 6% for nondiabetic

b. diabetic controlled should be less than 7%

89. Sengstaken-Blakemore tube prevents bleeding (esophageal varices)

a. Triple lumen

b. Have scissors at the bedside

c. Provide oral and nasal care every 3 hours

d. Used to reduce bleeding

90. Planning rehabilitation for a stroke patient

a. Assess functional status before developing plan

i. Walking, speaking, eating, ADLs

91. Cranial nerve II: Snellen test

92. T2-T3: Paraplegia

93. ICP: no lumbar puncture

94. A nurse observing a close chest tube drainage system is postop 1 day

thoracotomy? Continue bubbling in the suction chamber?

A. check the control outlet against the wall

B. observe all the connection tubing

C. Continue to monitor client respiratory status

D. Notify MD of the oxidation

95. A client admit to hospital report recurrent flank pain, nausea, and vomiting

within 24 hours. Which of the following priority nursing action?

A. Administered pain medication

B. Monitor intake and output

C. Administered antiemetics

D. Strain urine

96. A nurse is caring for client who have type 1 DM. The nurse misread client morning

blood glucose level at 210 mg/dL instead of 120 mg/dL base on this error. She

admitted insulin dose of 200 mg/dL before client breakfast. Which of the nursing

priority?

Monitor client for hypoglycemia

97. A nurse is caring a client who is schedule of colonoscopy. The client ask the

nurse if there will be a lot of pain during procedure. Which of the following is

appropriate nursing response?

A. No, you shouldn’t feel any pain because your rectum will be anesthesia

B. You may be sedated but you will feel discomfort during the instrument

insertion

C. Don’t worry ??

D. ???

98. A nurse is performing teaching for client who have recently diagnosis type 2 DM.

nurse should recognize that the client understood the teaching. Identifyhypoglycemia?

Select all

A. Moist, clammy skin

B. Tachycardia

C. Polyuria

D. Polydipsia

E. Polyphagia

F. ???

99. A nurse admitted morphine 2 mg IV push after client report pain and evaluate

client 15 min. later injection. Which follow adverse effect?

A . pain scale level of 6 to 4

B. sleepy but arouse when name call

C. O2 sat 94%

D. RR 8 bpm

100.A nurse assess a client who 8 score using the Glasgow coma scale to elevate of

consciousness. Describe the score.

A. Reflex alert client

B. Need of total nursing caring

C. Client in deep coma

D. Stable neurological status

101. client low sodium diet and reduce fluid intake to choose lunch.

A. Tuna sandwich on wheat bread, can of cocktail fruit, salad, and soda

B. Grill chicken sandwich on white bread, apple, salad, and ice tea

C. Grill cheese sandwich, tomato soup

D. Ham and bean

102.client acute MI. a cardiac enzyme obtain. Cardiac enzyme identify?

A. damage to the myocardial

B. determine the size MI

C. help to determine the location MI

D. ????

104. administered DDAVP to client diagnosis DI. Therapeutic effect

1. Specific gravity (1.015)

Ati MEDSURG PROCTORED 2019

A nurse in an emergency department is preparing to perform an ocular irrigation for a

client. Which of the following actions should the nurse plan to take?

a. Assess the client’s visual acuity prior to irrigation

b. Have the client turn their head toward the unaffected eye

c. Hold the irrigator syringe 3.81 cm (1.5 in) above the eye

d. Perform the irrigation with sterile water for irrigation

A nurse is preparing to administer lactated ringer’s via continuous IV infusion at 200

ml/hr. The IV tubing has a drop factor of 10 drops/ml. How many gtt/min should the

nurse set the IV pump to administer? Round to near whole number

33 gtt/min

A nurse is providing discharge teaching to a client who has a new prescription for

sublingual nitroglycerin. Which of the following client statements indicates an

understanding of the teaching?

a. I can keep my medications for 1 year before replacing it

b. I should lie down when I take this medication

c. I should discontinue this medication if I develop a headache

d. I can take up to five tablets in 15 minutes before seeking medical attention

A nurse is providing discharge teaching to an older adult client following a left total hip

arthroplasty. Which of the following instructions should the nurse include in the

teaching?

a. Clean the incision daily with hydrogen peroxide

b. You can cross your legs the ankles when sitting down

c. You should use an incentive spirometer every 8 hours

d. Install a raised toilet seat in your bathroom

A nurse is planning care for a client following a cardiac catheterization. Which of the

following actions should the nurse take?

a. Keep the client on bed rest for 24 hours

b. Limit the client’s fluid intake to 1 l per day

c. Maintain the client’s affected extremity in extension

d. Change the client’s dressing every 8 hour

A nurse is caring for a client who has a lower extremity fracture and a prescription for

crutches. Which of the following client statements indicates that the client is adapting to

their role change?

a. I will need to have my partner take over shopping for groceries and cooking the

meals for us

b. These crutches will make it impossible to care for my child

c. I feel bad that I have to ask my partner to keep the house clean

d. Its going to be difficult to tell my parents I cant take them to their appointments

anymore

A nurse is caring for a client who has gastroenteritis. Which of the following assessment

findings should the nurse recognize as an indication that the client is experiencing

dehydration?

a. Pitting, dependent edema

b. Distended jugular veins

c. Increased BP

d. Decreased BP

A nurse is caring for a client who has a contusion of the brainstem and reports thirst.

The client’s urinary output was 4,000 ml over the past 24 hour. The nurse should

anticipate a prescription for which of the following IV medication?

a. Desmopressin

b. Epinephrine

c. Furosemide

d. Nitroprusside

A nurse in a clinic receives a phone call from a client who recently started therapy with

an ACE inhibitor and reports a nagging dry cough. Which of the following responses by

the nurse is appropriate?

a. “your cough may require that you stop or change your medication”

b. “Increasing your daily fluid intake may eliminate your cough”

c. “sucking on lozenge may reduce the frequency of your cough”

d. You cough should go away in time”

A nurse is taking an admission history from a client who reports Raynaud’s disease.

Which of the following assessment findings should the nurse identify as a potential

trigger for exacerbations

of Raynaud’s?

a. Eating a strict vegetarian diet

b. A history of herpes zoster

c. Taking amiodipine for hypertension

d. Using a nicotine transdermal patch

A nurse is caring for a client who has a central venous access device and notes the

tubing has become disconnected. The client develops dyspnea and tachycardia. Which

of the following

actions should the nurse take first?

a. Perform an ECG

b. Obtain ABG values

c. Turn the client to his left side

d. Clamp the catheter

A nurse is completing an assessment of an older adult client and notes reddened areas

over the bony prominences, but the client’s skin is intact. Which of the following

interventions should the nurse include in the plan of care?

a. Turn and reposition the client every 4 hr

b. Apply an occlusive dressing

c. Support bony prominences with pillows

d. Massage the reddened areas three times a day

A home health nurse is making an initial visit to a client who has multiple sclerosis.

Which of the following actions is the priority for the nurse to take?

a. Discuss recommendations for eating and swallowing techniques

b. List strategies for family coping when dealing with possible role changes

c. Review the use of adaptive grooming devices to promote client independence

d. Give the client information about the local national multiple sclerosis society

A nurse in the emergency department is assessing a client. Which of the following

actions should the nurse take first? Exhibit

a. Obtain a sputum sample for culture

b. Administer ondansetron

c. Initiate airborne precautions

d. Prepare the client for a chest x-ray

A nurse is reviewing the medical record of a client to identify risk factors for colorectal

cancer. The nurse should identify which of the following findings as increasing the

client’s risk?

a. History of Crohn’s disease

b. BMI of 24

c. Diet high in fiber

d. Age 46 years

A nurse is caring for a client who is scheduled for a mastectomy. The client tells the

nurse, “I’m not sure I want to have a mastectomy.” Which of the following statements

should the nurse make?

a. “I can give you a list of other people who had the same procedure”

b. “You will be cancer-free if you have the procedure”

c. “I can give you additional information about the procedure”

d. “You should should get a second opinion regarding the procedure”

A nurse is preparing to administer a unit of packed RBCs to a client who is anemic.

Identify the sequence of steps the nurse should follow.

a. Obtain venous access using 19-gauge needle

b. Obtain the unit of packed RBCs from blood bank

c. Verify blood compatibility with another nurse

d. Initiate transfusion of the unit of packed RBCs

e. Remain with the client for the first 15 to 30 min of the infusion

A nurse is preparing a teaching plan for a client who has mucositis related to

chemotherapy treatment. Which of the following instructions should the nurse include?

a. “rinse your mouth with hydrogen peroxide”

b. “brush your teeth for 60 seconds twice daily”

c. “wear your dentures only during meals”

d. “floss your teeth following each meals”

A critical care nurse is assessing a client who has severe head injury. In response to

painful stimuli, the client does not open her eyes, displays decerebrate posturing, and

makes incomprehensible sounds. Which of the following Glasgow Coma Scale scores

should the nurse assign the client?

a. 5

b. 2

c. 13

d. 10

A nurse is providing discharge teaching to a client who has heart failure and instructs

him to limit sodium intake to 2 g per day. Which of the following statements by the client

indicates an understanding of the teaching?

a. “I can season my foods with garlic and onion salts”

b. “I can have mayonnaise on my sandwiches”

c. “I can have a frozen fruit juice bar for dessert”

d. “I can drink vegetable juice with a meal”

A nurse is preparing to perform ocular irrigation for a client following chemical splash to

the eye. Which of the following actions should the nurse plan to take first?

a. Instill 0.9% sodium chloride solution into the affected eye

b. Administer proparacaine eyedrops into the affected eye

c. Collect information about the irritant that caused the injury

A nurse is assessing a client following extubation from a ventilator. For which of the

following findings should the nurse intervene immediately?

a. Rhonchi

b. SaO2 92%

c. Sore throat

d. Stridor

A nurse is reviewing the laboratory reports of a client who has acute pancreatitis. Which

of the following findings should the nurse expect?

a. Elevated serum calcium

b. Elevated blood glucose

c. Decreased serum amylase

d. Decreased erythrocyte sedimentation rate

A nurse is reviewing the medical record of a client who has diabetes insipidus. Which of

the following findings should the nurse expect?

a. Hypothermia

b. Urine specific gravity 1.001 (<1.005)

c. Elevated blood pressure

d. BUN 15 mg/dl

A nurse is planning care for a client who has pulmonary embolism. Which of the

following interventions should the nurse include?

a. Initiate a continuous IV heparin infusion

b. Instruct the client to massage the lower extremities

c. Position the client on the left side

d. Measure vital signs every 4 hour

A nurse is providing discharge teaching to a client who is recovering from a sickle cell

crisis. Which of the following instructions should the nurse include?

a. Avoid extremely hot or cold temperatures

b. Limit fluids to 1.5 L per day

c. Limit alcohol intake to one drink per day

d. Avoid getting a flu vaccination

A nurse in the emergency department is caring for a client who is in hypovolemic shock.

Which of the following actions should the nurse take first?

a. Obtain a blood specimen for type and crossmatch

b. Insert a large-bore IV catheter

c. Administer IV therapy

d. Monitor urine output

A nurse is caring for a client who has an arteriovenous graft. Which of the following

findings indicates adequate circulation of the graft?

a. Dilated appearance of the graft

b. Absence of a bruit

c. Normotensive blood pressure

d. Palpable thrill

A nurse is assessing a client who has heart failure and is receiving a loop diuretic.

Which of the following findings indicates hypokalemia?

a. Oliguria

b. Hypertension

c. Muscle weakness

d. Positive chvostek’s sign (CHEEK)

A nurse is caring for a client who has a full-thickness burn injury covering 15% of their

body. Which of the following actions should the nurse take?

a. Weigh the client once per week

b. Provide the client with a protein intake of 1g/kg/day

c. Maintain a daily count of the client’s calorie intake

d. Place the client on a low-carb diet

A nurse is providing discharge teaching to a client who has an ileostomy. Which of the

following client statements indicates an understanding of the teaching?

a. “I will expect my stools to be loose”

b. “I will eat a high fiber diet’

c. “I will take a laxative when I’m constipated”

d. “I will empty my bag when it is full”

A nurse is caring for a client who is receiving total parental nutrition through a central

line. The current bag is nearly empty, and a new bag is unavailable from the pharmacy.

Which of the following actions should the nurse take?

a. Switch the infusion to a 10% dextrose solution

b. Discontinue the infusion and flush the line

c. Decrease the rate of infusion to last until the new bag is available

d. Start an infusion of 0.45% sodium chloride solution

A nurse is caring for a client who is 6 hr postoperative following a thyroidectomy. The

client reports tingling and numbness in the hands. The nurse should identify this as a

sign of which of the following electrolyte imbalances?

a. Hypocalcemia

b. Hypokalemia

c. Hypermagnesemia

d. Hypernatremia

A nurse is caring for a client who is caregiver for a relative who has chronic disease.

Which of the following statements indicates the client is adapting to the role change?

a. “I had to reschedule my doctor’s appointment last week”

b. “I have lunch with my friends once a week”

c. “I’ve lost 15 pounds in the past 2 months”

d. “I need to get my blood pressure medicine refilled”

A nurse is reviewing medications taken at home with a client who has angina. Which of

the following statements by the client indicates an understanding of the teaching?

a. “I should withhold my metoprolol if my heart rate is above 100 bpm”

b. “I should take my daily aspirin on an empty stomach”

c. “I should lie down before taking dose of isosorbide dinitrate”

d. “I should place a nitroglycerin tablet under my tongue every 10 minutes for up to four

doses”

A nurse in the post-anesthesia care unit is assessing a client following an

appendectomy and finds a 2-cm (3/4in) area of blood on the postoperative dressing.

Which of the following actions should the nurse take?

a. Apply pressure

b. Loosen the dressing

c. Circle the drainage

d. Apply a new dressingDocument continues below

Discover more from:

Medical And Surgical Nursing I (NUR 27)(NUR 27)

51 documentsGo to course

A nurse is caring for a client who is receiving mechanical ventilation. Which of following

interventions should the nurse implement?

a. Empty water from the ventilator tubing daily

b. Suction the client’s airway every 4 hour

c. Maintain the client in supine position

d. Perform oral care every 2 hour

A nurse is a planning care for a client who has full-thickness burns on the lower

extremities. Which of the following interventions should the nurse include?

a. Apply new gloves when alternating between wound care sites

b. Provide a diet of fresh fruits and vegetables for the client

c. Limit visitation time for the client’s children to 40 min per day

d. Clean the equipment in the client’s room once per week

A nurse is providing teaching for a client who has tuberculosis and a new prescription

for pyrazinamide. The nurse should instruct the client to notify the provider if which of

the following adverse effects occurs?

a. Hair loss

b. Polyuria

c. Weight gain

d. Jaundice

A nurse is planning care for a client who has left-sided hemiplegia following a stroke.

Which of the following actions should the nurse include in the plan of care?

a. Position the bedside table on the client’s left side

b. Place the plate guard on the client’s meal tray

c. Provide the client with a short handled reacher

d. Remind the client to use a cane on left side while ambulating

A nurse is performing an ear irrigation for a client. Which of the following actions should

the nurse take?

a. Use a cool fluid for irrigation

b. Insert the tip of the syringe 2.5cm (1in) into the ear canal

c. Tilt the client’s head 45 degrees

d. Point the tip of the syringe toward the top of the ear canal

A nurse is caring for a client who has a history of chemotherapy-induced nausea and

vomiting. Which of the following medications should the nurse administer prior to

chemotherapy?

a. Ondansetron

b. Sertraline

c. Methylprednisolone

d. Diphenhydramine

43. A nurse is preparing to discharge a client who has halo device and is reviewing new

prescriptions from the provider. The nurse should clarify which of the following

prescriptions with the provider?

a. Increase intake of fiber-rich foods

b. May place a small pillow under head when sleeping

c. May operate a motor vehicle when no longer taking analgesics

d. Take a tub bath instead of showers

A nurse is providing discharge teaching to a client who has tuberculosis. Which of the

following information should the nurse include in the teaching?

a. “You should wear an N95 respirator mask when you are at home”

b. “you will need to return in 2 weeks to provide a sputum specimen”

c. “You can drink alcohol after the first 6 weeks of treatment”

d. “Your provider will discontinue your medications after 3 months of therapy”

A nurse is assessing a client who has left-sided heart failure. Which of the following

findings should the nurse expect?

a. Flushed skin

b. Frothy sputum/Hacking cough

c. Jugular vein distention

d. Bradycardia

A nurse is planning care for a client who has osteoarthritis of the knees. Which of the

following interventions should the nurse include in the plan?

a. Avoid using a topical salicylate cream

b. Administer acetaminophen for pain management

c. Place a large pillow under the client’s knees when resting

d. Apply an ice pack directly to client’s knees

A nurse is caring for a client who is receiving continuous bladder irrigation following a

transurethral resection of the prostate (TURP). The client reports sharp lower abdominal

pain. Which of the following actions should the nurse first take?

a. Increase the client’s fluid intake

b. Reposition the client in bed

c. Check the client’s urine output

d. Administer PRN pain medication

A nurse is caring for a client who has Parkinson’s disease and is prescribed a level 1

dysphagia diet. Which of the following items should the nurse remove from the client’s

tray?

a. Vanilla milkshake

b. Peanut butter

c. Chocolate pudding

d. Applesauce

A nurse in a provider’s office is teaching a client about the self-management of GERD.

Which of the following instructions should the nurse include?

a. “eat a light meal 1 hour before bedtime”

b. “sleep with head of your bed elevated 6 inches”

c. “increase your caloric intake by 250 calories per day”

d. “lie down for 30 min after each meal”

A nurse is caring for a client who is postoperative following a partial thyroidectomy.

Which of the following findings is the priority for the nurse to report to the provider?

a. Client report of pain at the incision site

b. High-pitched sound on inspiration

c. Hypoactive bowel sounds

d. Loose tracheal secretions

A nurse is caring for a client who is 2 days postoperative following a below-the-knee

amputation and asks about the purpose of maintaining an elastic bandage around the

residual limb of the extremity. Which of the following is an appropriate response by the

nurse?

a. “the elastic bandage will prevent a post-op wound infection”

b. “the elastic bandage will prevent excessive edema”

c. “the elastic bandage will keep the sutures from loosening”

d. “the elastic bandage will keep you from seeing the surgical site”

A nurse is planning care for a client who is 8 hour post-op following a coronary artery

bypass grafting. Which of the following assessments should the nurse plan to perform

first?

a. Examine the surgical incision for drainage

b. Auscultate breath sounds

c. Palpate pulses distal to the graft donor site

d. Measure the client’s core body temperature

A nurse is providing instructions to a client who has primary syphilis. Which of the

following instructions should the nurse include in the discharge plan?

a. “you will need cryotherapy for 1 to 2 weeks”

b. “you will need to take an antiviral medication for 6 months”

c. “you will need 3 follow-up blood tests within 24 month period”

d. “you will need to be monitored for 15 minutes after receiving each medication dose”

A nurse is caring for a client who has hypotension, cool clammy skin, tachycardia, and

tachypnea. In which of the following positions should the nurse place the client?

a. Reverse Trendelenburg

b. Feet elevated

c. High fowlers’

d. Side lying

A nurse is teaching a client how to use a quad cane for ambulation following a right-

hemispheric stroke. Which of the following client actions indicates an understanding of

the teaching?

a. Client takes a step before advancing the cane

b. Client holds the cane with the left hand

c. Client moves the cane 2 feet ahead

d. Client advances the weaker leg forward first

A nurse is providing discharge teaching for a client who has new tracheostomy. Which

of the following statements by the client indicates an understanding of the teaching?

a. “ill remove the soiled tracheostomy ties prior to cleansing my stoma”

b. “ill cut a slit in a clean gauze pad to use as a stoma dressing”

c. “ill insert the obturator after cleaning my stoma”

d. “ill cleanse the cannula with half-strength hydrogen peroxide”

A nurse is preparing to administer furosemide to a client who has acute heart failure.

Which of the following laboratory results should the nurse identify as contraindications

for receiving the medications?

a. BUN 18 mg/dl

b. Creatinine 0.8 mg/dl

c. Potassium 3.2 mEq/l

d. Sodium 136 meEq/l

A nurse is caring for a client admitted with a skull fracture. Which of the following

assessment findings should be of greatest concern to the nurse?

a. Bilateral pupil diameter changes from 4 to 2 mm

b. WBC count changes from 9,000 to 16,000/mm3

c. Pulse pressure changes from 30 to 20 mm Hg

d. Glasgow Coma Scale score changes from 14 to 9

A nurse is assessing a client who has myasthenia gravis. Which of the following client

statements should indicate to the nurse that the client needs a referral for occupational

therapy?

a. “I have a hard time with brushing my hair”

b. “I would rather be in a wheelchair than use a walker to get around”

c. “I’ve been having problems with bladder control”

d. “I have difficulty swallowing food”

A nurse is providing discharge teaching to a client who will be self-administering insulin

at home. Which of the following information should the nurse include regarding needle

disposal?

a. “secure the cap tightly over the needle before you discard it”

b. “remove the needle from the syringe before you place it in the trash”

c. “you can discard needles in an empty bleach bottle with a lid”

d. “place your storage container in a recycle bin when it is full”

A nurse is assessing a client who has arteriovenous (AV) graft in the left forearm. Which

of the following findings should indicate to the nurse a complication of vascular access?

a. 2+ left radical pulse

b. Absence of a bruit

c. Presence of a palpable thrill

d. Dilated appearance of the AV site

A client who is deaf and communicates using sign language is being admitted by a

nurse who does not know sign language. Which of the following actions should the

nurse take?

a. Familiarize themselves with commonly used sign language

b. Ask a family member to be present during the admission

c. Obtain a board that uses colored pictures as communication

d. Request an interpreter during the initial assessment

A nurse is planning care for a client who has unrepaired intertrochanteric fracture and

has Buck’s traction placed to the affected leg. Which of the following interventions

should the

nurse include?

a. Situate the client’s heel in the heel of the traction boot

b. Apply weights of the traction to total 9.1 kg (20lb)

c. Place the footplate against the foot of the bed

d. Remove the boot for skin inspection every 12 hours.

A nurse is caring for a client who sustained a spinal cord injury in a diving accident.

Which of the following actions should the nurse take?

a. Assess the client’s neurological status every 8 hour

b. Monitor urine output hourly

c. Provide the client with a low-fiber diet

d. Log roll the client every 4 hour

A nurse is planning care for a client who has a central venous access device for

intermittent infusions. Which of the following actions should the nurse include in the plan

of care?

a. Flush the catheter using a 10ml syringe

b. Change the dressing every 24 hour

c. Use clean technique when changing the dressing

d. Cleanse the site with povidone-iodine

A nurse in the emergency department is caring for a client who has a gunshot wound to

the abdomen. Which of the following actions should the nurse take first?

a. Check the color of the client’s skin

b. Remove all of the client’s clothing

c. Administer an opioid analgesic

d. Prepare the client for periorbital lavage

A nurse is caring for a client following a bronchoscopy. Which of the following actions

should the nurse take first?

a. Check the client’s gag reflex

b. Inform the client they might experience a low-grade fever

c. Instruct the client to report bleeding

d. Provide the client with sips of water

A nurse is developing a plan of care for a client who is returning from the PACU

following a left below-the-knee amputation. Which of the following interventions should

the nurse include in the plan?

a. Provide the client with a firm mattress

b. Wrap the client’s residual limb with elastic bandage in a distal to proximal direction

c. Place the client’s residual limb in a dependent position when possible

d. Keep the client in a supine position for 48 hours

A nurse is instructing a client who has a new diagnosis of type 1 diabetes mellitus about

the sick-day rules. Which of the following statements by the client indicates an

understanding of

the teaching?

a. “I will monitor my blood glucose every 8 hours”

b. “I will consume 250 grams of carbs daily while I’m sick”

c. “I will not take my diabetes medications while I am sick”

d. “I will check urine for ketones if my blood glucose is greater than 240 mg/dl

A nurse is reviewing ABG results for a client who has COPD. Which of the following

findings should the nurse expect?

a. pH 7.38

b. PaO2 85 mm Hg

c. PaCO2 48 mm Hg

d. HCO3- 25 mEq/l

A nurse is admitting a client to a medical unit following placement of a permanent

pacemaker. Which of the following findings requires further assessment by the nurse?

a. Sneezing

b. Hiccups

c. Presence of a sharp spike prior to the QRS complex on the ECG

d. Presence of intrinsic P waves following a QRS complex on the ECG

A nurse is caring for a client who experienced extensive burns to the arms and torso.

Which of the following actions should the nurse take regarding the client’s oral

nutritional intake?

a. Adhere to scheduled meal times three times daily

b. Encourage the client to eat as many calories as possible

c. Limit the client’s fluid intake to 1,500 ml/day

d. Avoid the use of supplemental feedings throughout the day

A nurse is planning care for a client who is 1 day postoperative following an open

cholecystectomy. Which of the following interventions should the nurse include in the

plan of

care?

a. Place pillows under the client’s knees

b. Apply compression stockings to the lower extremities

c. Avoid use of anticoagulants

d. Discourage leg exercises while in bed

A nurse is caring for a client who has duodenal ulcer. Which of the following actions

should the nurse take? Exhibit

a. Restrict the client’s fluid intake to 1,000 ml/day

b. Infuse packed RBCs

c. Administer the client’s naproxen prescription

d. Offer a snack before bedtime

A nurse is assessing an older adult client at a health fair. Which of the following

statements by the client is the nurse’s priority?

a. “I can’t seem to get reading materials far enough away to see the words”

b. “I’m having more difficulty telling the difference between blues and greens”

c. “I’ve noticed that there is a gray ring around the colored part of my eye”

d. “In the last day, I have had a severe headache and pain around my right eye”

A nurse is caring for an adolescent client who has an acute kidney injury. Which of the

following laboratory findings should the nurse anticipate?

a. BUN 8 mg/dl

b. Hgb 20 g/dl

c. Potassium 6.8 mEq/l

d. Creatinine 0.4 mg/dl

A nurse is planning care for an older adult client who has Meniere’s disease. Which of

the following interventions should the nurse include in the plan?

a. Perform range-of-motion exercises to the client’s neck every 4 hour

b. Limit the client’s fluid intake to 1,500 ml/day

d. Administer aspirin if the client reports a headache

c. Encourage the client to change positions slowly

A nurse is preparing to receive a client from surgery following a transverse colon

resection with colostomy placement. The nurse should expect to assess the stoma at

which of the following locations?

a. Upper left abdomen (THIS ONE IS A DIAGRAM CHOOSE THE POINT ON THE

TOP LEFT OF

ABDOMEN)

A nurse is admitting a client to the emergency department after a gunshot wound to the

abdomen. Which of the following actions should the nurse take to help prevent the

onset of acute kidney failure?

a. Initiate beta blocker therapy

b. Insert a urinary catheter

c. Prepare the client for an intravenous pyelogram

d. Administer IV fluids to the client

A nurse is preparing to administer 1 unit of packed RBCs to an adult client. Which of the

following actions should the nurse plan to take?

a. Administer through a 22-gauge IV catheter

b. Prime the IV tubing with 0.45% sodium chloride

c. Complete the transfusion within 2 hour

d. Slow the transfusion rate if the client reports itching

A nurse is planning care for a client who has developed nephrotic syndrome. Which of

the following dietary recommendations should the nurse include?

a. Increase phosphorus intake

c. Increase potassium intake

d. Decrease carbohydrate intake

b. Decrease protein intake

A nurse is caring for an older adult client who has dementia. Which of the following

questions should the nurse ask to assess the client’s abstract thinking?

a. “can you count backwards from 100 intervals of 7?”

b. “what is meant by the saying, don’t beat around the brush?

c. “what do you understand about your condition?”

d. “can you tell me the state where you were born?”

A nurse is caring for a client who has cervical cancer and is receiving brachytherapy.

Which of the following actions should the nurse take?

a. Keep the soiled bed linens in the client’s room

b. Instruct visitors to remain 3 feet from the client

c. Discard the radioactive device in the client’s trash can

d. Limit time for visitors to 2 hour per day

A nurse is preparing a client for a lumbar puncture. Which of the following images

indicates the position should the nurse assist the client into for this procedure?

a. Side-lying (THIS IS A DIAGRAM CHOOSE THE PERSON IN A FETAL POSITION)

A nurse is caring for a client who has cervical cancer and a sealed radiation implant.

Which of the following actions should the nurse take?

a. Place long-handled forceps at the client’s bedside

b. Attach a dosimeter badge to the client’s gown

c. Leave unused equipment in the client’s room until discharge

d. Move the client’s soiled linens to a designated container outside the room

A nurse is teaching a client who has Graves’ disease about recognizing the

manifestations of the thyroid storm. Which of the following findings should the nurse

include in the teaching?

a. Increased temperature

b. Decreased HR

c. Hypotension

d. Lethargy

A nurse is caring for a client who is postoperative following a complete thyroidectomy.

Which of the following findings is the priority for the nurse to report to the provider?

a. Serosanguineous drainage

b. Muscle twitching

c. Client report of nausea

d. Client report incisional pain

A nurse is reviewing ECG rhythm strips for a group of clients. The nurse should identify

that which of the following rhythms indicates bradycardia?

pick the brady strip

A nurse is caring for a client who is receiving epidural analgesics. Which of the following

assessment findings in the nurse’s priority?

a. Bladder distention

b. Hypoactive bowel sounds

c. Hypotension

d. Weakness to lower extremities

A nurse is planning care for a client who has status epilepticus. Which of the following

interventions is the nurse’s priority to include?

a. Turn the client to the lateral position during seizure activity

b. Provide the client oxygen at 6 l/min using a nasal cannula

c. Administer phenytoin IV bolus to the client

d. Administer diazepam intravenously to the client

A nurse is caring for a client following a below-the knee amputation. The client states.

“my life is over.” Which of the following responses should the nurse make?

a. “you are upset. We can talk about this later?”

b. “would you like to meet with another client who is an amputee?”

c. “why do you think your life is over?”

d. “most people can adjust following this surgery”

A nurse in a clinic is providing preventive teaching to an older adult client during a well

visit. The nurse should instruct the client that which of the following immunizations are

recommended for healthy adults after the age of 60? Select All That Apply

a. Herpes zoster

b. Influenza

c. Meningococcal

d. Human papillomavirus

e. Pneumococcal polysaccharide

A nurse is planning care for a client who has dementia. Which of the following

interventions should the nurse plan to include?

a. Turn off all lights in the client’s room at night

b. Place the client’s bed at the lowest height

c. Request a prescription for a nightly sedative

d. Assist the client with toileting at least once every 4 hour

A nurse is assessing a client who has a new diagnosis of type 1 diabetes mellitus.

Which of the following findings indicates that the client is experiencing hypoglycemia?

a. Abdominal cramping

b. Increased perspiration

c. Dehydration

d. Fruity odor to breath

A nurse in the PACU is assessing a client who is postoperative following general

anesthesia. Which of the following findings is the priority to address?

b. Decreased body temperature

c. Indistinct, rambling speech

d. Piloerection of the skin

a. Vomiting upon arousal

A nurse is caring for a client who has hypervolemia. Which of the following is an

expected assessment finding?

a. Bradycardia

b. Hypotension

c. Loss of skin turgor

d. Weight gain

A nurse is teaching about measures to prevent recurring urinary tract infections with a

female client. Which of the following information should the nurse include in the

teaching? Select All That Apply

a. Take a warm bubble bath daily

b. Void every 6 hour during the day

c. Drink low-fructose cranberry juice

d. Wipe the perineal area from front to back after urinating

e. Drink 3L of fluids daily

A nurse is caring for a client following a cardiac catheterization who has hives and

urticaria following administration of IV contrast dye. Which of the following medications

should the nurse plan to administer?

a. Spironolactone

b. Desmopressin

c. Metoclopramide

d. Diphenhydramine

A home care nurse is planning to use nonpharmacological pain relief measures for an

older adult client who has severe chronic back pain. Which of the following guidelines

should the nurse use?

a. Discontinue opioids before trying nonpharmacological methods of pain relief

b. Use imagery with clients who have difficulty with focus and concentration

c. Distraction changes the client’s perception of pain, but does not affect the cause

d. Pain relief from the use of heat and cold continues for several hours after removal of

the

stimulus

A nurse is caring for a female client who is receiving total parental nutrition without fat

emulsion. Which of the following findings should the nurse report?

a. Crackles in the bilateral lung bases

b. Weight gain of 1.3 kg (3lb) over the past 7 days

c. Triglyceride 110 mg/dl

d. Bowel sounds absent in lower quadrants

A nurse in an emergency department is preparing to perform ocular irrigation for a client which of the following actions should the nurse plan to take?
Perform the irrigation with sterile water for irrigation

A nurse is preparing to administer lactated ringers via continuous IV infusion at 200 ml/h. The IV tubing has a drop factor of 10 drops/ml. How may gtts/min should the nurse set the IV pump at?
33 gtt/min

A nurse is providing discharge teaching to a client who has a new prescription for sublingual nitroglycerin. Which of the following client statements indicates an understanding of the teaching?
I should lie down when I take this medication

A nurse is providing discharge teaching to an older adult client following left total hip arthroplasty. Which of the following instructions should the nurse include in the teaching?
Install a raised toilet seat in your bathroom

A nurse is caring for a client who has a lower extremity fracture and is prescribed for crutches. Which of the following client statement indicates that the client is adapting to their role change
i will need to have my partner take shopping for groceries and cooking meals for us

A nurse is planning care for a client following a cardiac catheterization which of the following actions should the nurse take?
Maintain the client’s affected extremity in extension

A nurse is caring for a client who has gastroenteritis which of the following assessment findings should the nurse recognize as indication that the client is experiencing dehydration?
Decreased BP

A nurse is caring for a client who has a concussion of the brainstem and reports thirst the client’s urinary output was 4,000ml over the past 24 hours the nurse should anticipate a prescription for which of the following IV medication

A nurse in a client receives a phone call from a client who recently started therapy with an ACE inhibitor and reports a nagging dry cough. Which of the following responses by the nurse is appropriate?
“your cough may require that you stop or change your medication”

A nurse is taking an admission history from a client who reports Raynaud’s Disease which of the following assessment findings should the nurse identify as a potential trigger for exacerbations of Raynaud’s ?
Using a nicotine transdermal patch

A nurse is caring for a client who has a central venous access device and notes that the tubing has become disconnected the client develops dyspnea and tachycardia which of the following actions should the nurse take first?
Clamp the catheter

A nurse is completing an assessment of an older adult client and notes reddened areas over the bony prominences, but the client’s skin is intact. Which of the following interventions should the nurse include in the plan of care
Support bony prominences with pillows

A home health is making an initial visit to a client who has multiple sclerosis which of the following actions is the priority for the nurse to take?
Discuss recommendations for eating and swallowing techniques

A nurse in the emergency department is assessing a clients which of the following actions should the use take first?
Initiate airborne precautions

A nurse is reviewing the medical records of a clients to identify risk factors for colorectal cancer the nurse should identify which of the following findings as increasing the clients risk?
History of Crohn’s disease

A nurse is caring for a client who is schedule for a mastectomy the clients tell the nurse I’m not sure I want to have a mastectomy which of the following statements should the nurse make
I can give you additional information about the procedure

A nurse is preparing to administer a unit of packed RBC to a client who is anemic Identify the sequence of steps the nurse should follow?
Obtain venous access using 19 needle
Obtain the unit of packed RBCs from the blood bank
Verify blood compatibility with another nurse
Initiate transfusion of the unit of packed RBCs
Remain with the client for the first 15 and 30 mins of the infusion

A nurse is preparing a teaching plan for a client who has mucositis related to chemotherapy treatment which of the following instructions should the nurse include
Floss your teeth following each meals

A critical care nurse is assessing a client who has severe head injury. In response to painful stimuli, the client does not open her eye, displays decerebrate posturing, and makes incomprehensible sounds, which of the following GCS scores should the nurse assess the client
5

A nurse is providing discharge teaching to a client who as heart failure and instructs him to limit sodium intake to 2 g per day which of the following statements by the client indicates an understanding of the teaching?
I can have a frozen fruit juice bar for dessert

A nurse is preparing to perform ocular irrigation for a client following chemical splash to the eye which of the following actions should the nurse take first
Collect information about irritant that caused the injury

A nurse is assessing a client following extubation from a ventilator. For which of the following findings should the nurse intervene immediately
Stridor

A nurse is reviewing the laboratory reports of a client who as acute pancreatitis which of the following findings should the nurse expect?
Elevated blood glucose

A nurse is reviewing the medical record of a client who has diabetes insipidus which of the following finding should the nurse expect
Urine specific gravity 1.001

A nurse is planning care for a client who has a pulmonary embolism which of the following interventions should the nurse include?
Initiate a continuous IV heparin infusion

A nurse is providing discharge teaching to a client who is recovering from a sickle cell crisis which of the following instructions should the nurse include?
Avoid extremely hot or cold temp

A nurse in the emergency department is caring for a client who is in hypovolemic shock of the following actions should the nurse take first
Insert a large bore IV

A nurse is caring for a client who has a full-thickness burn injury covering 15% of their body which of the following actions should the nurse take?
Maintain a daily count of the client’s calorie intake

A nurse is caring for a client who has an arteriovenous graft which of the following findings indicates adequate circulation of the graft?
Palpable thrill

A nurse is assessing a client who has heart failure and is receiving a loop diuretic which of the following findings indicates hypokalemia?
Muscle Weakness

A nurse is providing discharge teaching to a client who has an ileostomy which of the following clients statement indicates an understanding of the teaching
I will expect my stool to be loose

A nurse is caring for a client who is receiving total parental nutrition through a central line the current bag is nearly empty and a new bag is unavailable from the pharmacy which of the following actions should the nurse take
Switch the infusion to a 10% dextrose solution

A nurse is caring for a client who is 6 hr postoperative following a thyroidectomy the client reports tingling and numbness in the hands the nurse should identify this as a sign of which of the following electrolyte imbalance
Hypocalcemia

A nurse is caring for a client who is caregiver for a relative who has chronic diseases which of the following statement indicates the client is adapting to the role change
I need to get my blood pressure medicine refilled

A nurse is reviewing medication taken at home with a client who has angina which of the following statements by the client indicates an understanding of the teaching
I should lie down before taking dose of isosorbide denigrate

A nurse is the post-anesthesia care unit is assessing a client following an appendectomy and finds a 2 cm area of blood on the postoperative dressing which of the following actions should the nurse take
circle the drainage

A nurse is caring for a client who is receiving mechanical

A nurse in an emergency department is preparing to perform an ocular irrigation for a client. Which of the following actions should the nurse plan to take?
a. Assess the client’s visual acuity prior to irrigation
b. Have the client turn their head toward the unaffected eye
c. Hold the irrigator syringe 3.81 cm (1.5 in) above the eye
d. Perform the irrigation with sterile water for irrigation
d. Perform the irrigation with sterile water for irrigation

A nurse is preparing to administer lactated ringer’s via continuous IV infusion at 200 ml/hr. The IV tubing has a drop factor of 10 drops/ml. How many gtt/min should the nurse set the IV pump to administer? Round to near whole number
33 gtt/min

A nurse is providing discharge teaching to a client who has a new prescription for sublingual nitroglycerin. Which of the following client statements indicates an understanding of the teaching?
a. I can keep my medications for 1 year before replacing it
b. I should lie down when I take this medication
c. I should discontinue this medication if I develop a headache
d. I can take up to five tablets in 15 minutes before seeking medical attention
b. I should lie down when I take this medication

A nurse is providing discharge teaching to an older adult client following a left total hip arthroplasty. Which of the following instructions should the nurse include in the teaching?
a. Clean the incision daily with hydrogen peroxide
b. You can cross your legs the ankles when sitting down
c. You should use an incentive spirometer every 8 hours
d. Install a raised toilet seat in your bathroom
d. Install a raised toilet seat in your bathroom

A nurse is planning care for a client following a cardiac catheterization. Which of the following actions should the nurse take?
a. Keep the client on bed rest for 24 hours
b. Limit the client’s fluid intake to 1 l per day
c. Maintain the client’s affected extremity in extension
d. Change the client’s dressing every 8 hour
c. Maintain the client’s affected extremity in extension

A nurse is caring for a client who has a lower extremity fracture and a prescription for crutches. Which of the following client statements indicates that the client is adapting to their role change?
a. I will need to have my partner take over shopping for groceries and cooking the meals for us
b. These crutches will make it impossible to care for my child
c. I feel bad that I have to ask my partner to keep the house clean
d. Its going to be difficult to tell my parents I cant take them to their appointments anymore
a. I will need to have my partner take over shopping for groceries and cooking the meals for us

A nurse is caring for a client who has gastroenteritis. Which of the following assessment findings should the nurse recognize as an indication that the client is experiencing dehydration?
a. Pitting, dependent edema
b. Distended jugular veins
c. Increased BP
d. Decreased BP
d. Decreased BP

A nurse is caring for a client who has a contusion of the brainstem and reports thirst. The client’s urinary output was 4,000 ml over the past 24 hour. The nurse should anticipate a prescription for which of the following IV medication?
a. Desmopressin
b. Epinephrine
c. Furosemide
d. Nitroprusside
a. Desmopressin

A nurse in a clinic receives a phone call from a client who recently started therapy with an ACE inhibitor and reports a nagging dry cough. Which of the following responses by the nurse is appropriate?
a. “your cough may require that you stop or change your medication”
b. “Increasing your daily fluid intake may eliminate your cough”
c. “sucking on lozenge may reduce the frequency of your cough”
d. You cough should go away in time”
a. “your cough may require that you stop or change your medication”

A nurse is taking an admission history from a client who reports Raynaud’s disease. Which of the following assessment findings should the nurse identify as a potential trigger for exacerbations
of Raynaud’s?
a. Eating a strict vegetarian diet
b. A history of herpes zoster
c. Taking amiodipine for hypertension
d. Using a nicotine transdermal patch
d. Using a nicotine transdermal patch

A nurse is caring for a client who has a central venous access device and notes the tubing has become disconnected. The client develops dyspnea and tachycardia. Which of the following
actions should the nurse take first?
a. Perform an ECG
b. Obtain ABG values
c. Turn the client to his left side
d. Clamp the catheter
d. Clamp the catheter

A nurse is completing an assessment of an older adult client and notes reddened areas over the bony prominences, but the client’s skin is intact. Which of the following interventions should the nurse include in the plan of care?
a. Turn and reposition the client every 4 hr
b. Apply an occlusive dressing
c. Support bony prominences with pillows
d. Massage the reddened areas three times a day
c. Support bony prominences with pillows

A home health nurse is making an initial visit to a client who has multiple sclerosis. Which of the following actions is the priority for the nurse to take?
a. Discuss recommendations for eating and swallowing techniques
b. List strategies for family coping when dealing with possible role changes
c. Review the use of adaptive grooming devices to promote client independence
d. Give the client information about the local national multiple sclerosis society
a. Discuss recommendations for eating and swallowing techniques

A nurse in the emergency department is assessing a client. Which of the following actions should the nurse take first? Exhibit
a. Obtain a sputum sample for culture
b. Administer ondansetron
c. Initiate airborne precautions
d. Prepare the client for a chest x-ray
c. Initiate airborne precautions

A nurse is reviewing the medical record of a client to identify risk factors for colorectal cancer. The nurse should identify which of the following findings as increasing the client’s risk?
a. History of Crohn’s disease
b. BMI of 24
c. Diet high in fiber
d. Age 46 years
a. History of Crohn’s disease

A nurse is caring for a client who is scheduled for a mastectomy. The client tells the nurse, “I’m not sure I want to have a mastectomy.” Which of the following statements should the nurse make?
a. “I can give you a list of other people who had the same procedure”
b. “You will be cancer-free if you have the procedure”
c. “I can give you additional information about the procedure”
d. “You should should get a second opinion regarding the procedure”
c. “I can give you additional information about the procedure”

A nurse is preparing to administer a unit of packed RBCs to a client who is anemic. Identify the sequence of steps the nurse should follow.
e. Remain with the client for the first 15 to 30 min of the infusion
a. Obtain venous access using 19-gauge needle
c. Verify blood compatibility with another nurse
d. Initiate transfusion of the unit of packed RBCs
b. Obtain the unit of packed RBCs from blood bank
a. Obtain venous access using 19-gauge needle
b. Obtain the unit of packed RBCs from blood bank
c. Verify blood compatibility with another nurse
d. Initiate transfusion of the unit of packed RBCs
e. Remain with the client for the first 15 to 30 min of the infusion

A nurse is preparing a teaching plan for a client who has mucositis related to chemotherapy treatment. Which of the following instructions should the nurse include?
a. “rinse your mouth with hydrogen peroxide”
b. “brush your teeth for 60 seconds twice daily”
c. “wear your dentures only during meals”
d. “floss your teeth following each meals”
d. “floss your teeth following each meals”

A critical care nurse is assessing a client who has severe head injury. In response to painful stimuli, the client does not open her eyes, displays decerebrate posturing, and makes incomprehensible sounds. Which of the following Glasgow Coma Scale scores should the nurse assign the client?
a. 5
b. 2
c. 13
d. 10
a. 5

A nurse is providing discharge teaching to a client who has heart failure and instructs him to limit sodium intake to 2 g per day. Which of the following statements by the client indicates an understanding of the teaching?
a. “I can season my foods with garlic and onion salts”
b. “I can have mayonnaise on my sandwiches”
c. “I can have a frozen fruit juice bar for dessert”
d. “I can drink vegetable juice with a meal”
c. “I can have a frozen fruit juice bar for dessert”

A nurse is preparing to perform ocular irrigation for a client following chemical splash to the eye. Which of the following actions should the nurse plan to take first?
a. Instill 0.9% sodium chloride solution into the affected eye
b. Administer proparacaine eyedrops into the affected eye
c. Collect information about the irritant that caused the injury
c. Collect information about the irritant that caused the injury

A nurse is assessing a client following extubation from a ventilator. For which of the following findings should the nurse intervene immediately?
a. Rhonchi
b. SaO2 92%
c. Sore throat
d. Stridor
d. Stridor

A nurse is reviewing the laboratory reports of a client who has acute pancreatitis. Which of the following findings should the nurse expect?
a. Elevated serum calcium
b. Elevated blood glucose
c. Decreased serum amylase
d. Decreased erythrocyte sedimentation rate
b. Elevated blood glucose

A nurse is reviewing the medical record of a client who has diabetes insipidus. Which of the following findings should the nurse expect?
a. Hypothermia
b. Urine specific gravity 1.001 (<1.005)
c. Elevated blood pressure
d. BUN 15 mg/dl
b. Urine specific gravity 1.001 (<1.005)

A nurse is planning care for a client who has pulmonary embolism. Which of the following interventions should the nurse include?
a. Initiate a continuous IV heparin infusion
b. Instruct the client to massage the lower extremities
c. Position the client on the left side
d. Measure vital signs every 4 hour
a. Initiate a continuous IV heparin infusion

A nurse is providing discharge teaching to a client who is recovering from a sickle cell crisis. Which of the following instructions should the nurse include?
a. Avoid extremely hot or cold temperatures
b. Limit fluids to 1.5 L per day
c. Limit alcohol intake to one drink per day
d. Avoid getting a flu vaccination
a. Avoid extremely hot or cold temperatures

A nurse in the emergency department is caring for a client who is in hypovolemic shock. Which of the following actions should the nurse take first?
a. Obtain a blood specimen for type and crossmatch
b. Insert a large-bore IV catheter
c. Administer IV therapy
d. Monitor urine output
b. Insert a large-bore IV catheter

A nurse is caring for a client who has an arteriovenous graft. Which of the following findings indicates adequate circulation of the graft?
a. Dilated appearance of the graft
b. Absence of a bruit
c. Normotensive blood pressure
d. Palpable thrill
d. Palpable thrill

A nurse is assessing a client who has heart failure and is receiving a loop diuretic. Which of the following findings indicates hypokalemia?
a. Oliguria
b. Hypertension
c. Muscle weakness
d. Positive chvostek’s sign (CHEEK)
c. Muscle weakness

A nurse is caring for a client who has a full-thickness burn injury covering 15% of their body. Which of the following actions should the nurse take?
a. Weigh the client once per week
b. Provide the client with a protein intake of 1g/kg/day
c. Maintain a daily count of the client’s calorie intake
d. Place the client on a low-carb diet
c. Maintain a daily count of the client’s calorie intake

A nurse is providing discharge teaching to a client who has an ileostomy. Which of the following client statements indicates an understanding of the teaching?
a. “I will expect my stools to be loose”
b. “I will eat a high fiber diet’
c. “I will take a laxative when I’m constipated”
d. “I will empty my bag when it is full”
a. “I will expect my stools to be loose”

A nurse is caring for a client who is receiving total parental nutrition through a central line. The current bag is nearly empty, and a new bag is unavailable from the pharmacy. Which of the following actions should the nurse take?
a. Switch the infusion to a 10% dextrose solution
b. Discontinue the infusion and flush the line
c. Decrease the rate of infusion to last until the new bag is available
d. Start an infusion of 0.45% sodium chloride solution
a. Switch the infusion to a 10% dextrose solution

A nurse is caring for a client who is 6 hr postoperative following a thyroidectomy. The client reports tingling and numbness in the hands. The nurse should identify this as a sign of which of the following electrolyte imbalances?
a. Hypocalcemia
b. Hypokalemia
c. Hypermagnesemia
d. Hypernatremia
a. Hypocalcemia

A nurse is caring for a client who is caregiver for a relative who has chronic disease. Which of the following statements indicates the client is adapting to the role change?
a. “I had to reschedule my doctor’s appointment last week”
b. “I have lunch with my friends once a week”
c. “I’ve lost 15 pounds in the past 2 months”
d. “I need to get my blood pressure medicine refilled”
d. “I need to get my blood pressure medicine refilled”

A nurse is reviewing medications taken at home with a client who has angina. Which of the following statements by the client indicates an understanding of the teaching?
a. “I should withhold my metoprolol if my heart rate is above 100 bpm”
b. “I should take my daily aspirin on an empty stomach”
c. “I should lie down before taking dose of isosorbide dinitrate”
d. “I should place a nitroglycerin tablet under my tongue every 10 minutes for up to four doses”
c. “I should lie down before taking dose of isosorbide dinitrate”

A nurse in the post-anesthesia care unit is assessing a client following an appendectomy and finds a 2-cm (3/4in) area of blood on the postoperative dressing. Which of the following actions should the nurse take?
a. Apply pressure
b. Loosen the dressing
c. Circle the drainage
d. Apply a new dressing
c. Circle the drainage

A nurse is caring for a client who is receiving mechanical ventilation. Which of following interventions should the nurse implement?
a. Empty water from the ventilator tubing daily
b. Suction the client’s airway every 4 hour
c. Maintain the client in supine position
d. Perform oral care every 2 hour
a. Empty water from the ventilator tubing daily

A nurse is a planning care for a client who has full-thickness burns on the lower extremities. Which of the following interventions should the nurse include?
a. Apply new gloves when alternating between wound care sites
b. Provide a diet of fresh fruits and vegetables for the client
c. Limit visitation time for the client’s children to 40 min per day
d. Clean the equipment in the client’s room once per week
a. Apply new gloves when alternating between wound care sites

A nurse is providing teaching for a client who has tuberculosis and a new prescription for pyrazinamide. The nurse should instruct the client to notify the provider if which of the following adverse effects occurs?
a. Hair loss
b. Polyuria
c. Weight gain
d. Jaundice
d. Jaundice

A nurse is planning care for a client who has left-sided hemiplegia following a stroke. Which of the following actions should the nurse include in the plan of care?
a. Position the bedside table on the client’s left side
b. Place the plate guard on the client’s meal tray
c. Provide the client with a short handled reacher
d. Remind the client to use a cane on left side while ambulating
b. Place the plate guard on the client’s meal tray

A nurse is performing an ear irrigation for a client. Which of the following actions should the nurse take?
a. Use a cool fluid for irrigation
b. Insert the tip of the syringe 2.5cm (1in) into the ear canal
c. Tilt the client’s head 45 degrees
d. Point the tip of the syringe toward the top of the ear canal
d. Point the tip of the syringe toward the top of the ear canal

A nurse is caring for a client who has a history of chemotherapy-induced nausea and vomiting. Which of the following medications should the nurse administer prior to chemotherapy?
a. Ondansetron
b. Sertraline
c. Methylprednisolone
d. Diphenhydramine
a. Ondansetron

  1. A nurse is preparing to discharge a client who has halo device and is reviewing new prescriptions from the provider. The nurse should clarify which of the following prescriptions with the provider?
    a. Increase intake of fiber-rich foods
    b. May place a small pillow under head when sleeping
    c. May operate a motor vehicle when no longer taking analgesics
    d. Take a tub bath instead of showers
    c. May operate a motor vehicle when no longer taking analgesics

A nurse is providing discharge teaching to a client who has tuberculosis. Which of the following information should the nurse include in the teaching?
a. “You should wear an N95 respirator mask when you are at home”
b. “you will need to return in 2 weeks to provide a sputum specimen”
c. “You can drink alcohol after the first 6 weeks of treatment”
d. “Your provider will discontinue your medications after 3 months of therapy”
b. “you will need to return in 2 weeks to provide a sputum specimen”

A nurse is assessing a client who has left-sided heart failure. Which of the following findings should the nurse expect?
a. Flushed skin
b. Frothy sputum/Hacking cough
c. Jugular vein distention
d. Bradycardia
b. Frothy sputum/Hacking cough

A nurse is planning care for a client who has osteoarthritis of the knees. Which of the following interventions should the nurse include in the plan?
a. Avoid using a topical salicylate cream
b. Administer acetaminophen for pain management
c. Place a large pillow under the client’s knees when resting
d. Apply an ice pack directly to client’s knees
b. Administer acetaminophen for pain management

A nurse is caring for a client who is receiving continuous bladder irrigation following a transurethral resection of the prostate (TURP). The client reports sharp lower abdominal pain. Which of the following actions should the nurse first take?
a. Increase the client’s fluid intake
b. Reposition the client in bed
c. Check the client’s urine output
d. Administer PRN pain medication
c. Check the client’s urine output

A nurse is caring for a client who has Parkinson’s disease and is prescribed a level 1 dysphagia diet. Which of the following items should the nurse remove from the client’s tray?
a. Vanilla milkshake
b. Peanut butter
c. Chocolate pudding
d. Applesauce
b. Peanut butter

A nurse in a provider’s office is teaching a client about the self-management of GERD. Which of the following instructions should the nurse include?
a. “eat a light meal 1 hour before bedtime”
b. “sleep with head of your bed elevated 6 inches”
c. “increase your caloric intake by 250 calories per day”
d. “lie down for 30 min after each meal”
b. “sleep with head of your bed elevated 6 inches”

A nurse is caring for a client who is postoperative following a partial thyroidectomy. Which of the following findings is the priority for the nurse to report to the provider?
a. Client report of pain at the incision site
b. High-pitched sound on inspiration
c. Hypoactive bowel sounds
d. Loose tracheal secretions
b. High-pitched sound on inspiration

A nurse is caring for a client who is 2 days postoperative following a below-the-knee amputation and asks about the purpose of maintaining an elastic bandage around the residual limb of the extremity. Which of the following is an appropriate response by the nurse?
a. “the elastic bandage will prevent a post-op wound infection”
b. “the elastic bandage will prevent excessive edema”
c. “the elastic bandage will keep the sutures from loosening”
d. “the elastic bandage will keep you from seeing the surgical site”
b. “the elastic bandage will prevent excessive edema”

A nurse is planning care for a client who is 8 hour post-op following a coronary artery bypass grafting. Which of the following assessments should the nurse plan to perform first?
a. Examine the surgical incision for drainage
b. Auscultate breath sounds
c. Palpate pulses distal to the graft donor site
d. Measure the client’s core body temperature
b. Auscultate breath sounds

A nurse is providing instructions to a client who has primary syphilis. Which of the following instructions should the nurse include in the discharge plan?
a. “you will need cryotherapy for 1 to 2 weeks”
b. “you will need to take an antiviral medication for 6 months”
c. “you will need 3 follow-up blood tests within 24 month period”
d. “you will need to be monitored for 15 minutes after receiving each medication dose”
c. “you will need 3 follow-up blood tests within 24 month period”

A nurse is caring for a client who has hypotension, cool clammy skin, tachycardia, and tachypnea. In which of the following positions should the nurse place the client?
a. Reverse Trendelenburg
b. Feet elevated
c. High fowlers’
d. Side lying
b. Feet elevated

A nurse is teaching a client how to use a quad cane for ambulation following a right-hemispheric stroke. Which of the following client actions indicates an understanding of the teaching?
a. Client takes a step before advancing the cane
b. Client holds the cane with the left hand
c. Client moves the cane 2 feet ahead
d. Client advances the weaker leg forward first
d. Client advances the weaker leg forward first

A nurse is providing discharge teaching for a client who has new tracheostomy. Which of the following statements by the client indicates an understanding of the teaching?
a. “ill remove the soiled tracheostomy ties prior to cleansing my stoma”
b. “ill cut a slit in a clean gauze pad to use as a stoma dressing”
c. “ill insert the obturator after cleaning my stoma”
d. “ill cleanse the cannula with half-strength hydrogen peroxide”
c. “ill insert the obturator after cleaning my stoma”

A nurse is preparing to administer furosemide to a client who has acute heart failure. Which of the following laboratory results should the nurse identify as contraindications for receiving the medications?
a. BUN 18 mg/dl
b. Creatinine 0.8 mg/dl
c. Potassium 3.2 mEq/l
d. Sodium 136 meEq/l
c. Potassium 3.2 mEq/l

A nurse is caring for a client admitted with a skull fracture. Which of the following assessment findings should be of greatest concern to the nurse?
a. Bilateral pupil diameter changes from 4 to 2 mm
b. WBC count changes from 9,000 to 16,000/mm3
c. Pulse pressure changes from 30 to 20 mm Hg
d. Glasgow Coma Scale score changes from 14 to 9
d. Glasgow Coma Scale score changes from 14 to 9

A nurse is assessing a client who has myasthenia gravis. Which of the following client statements should indicate to the nurse that the client needs a referral for occupational therapy?
a. “I have a hard time with brushing my hair”
b. “I would rather be in a wheelchair than use a walker to get around”
c. “I’ve been having problems with bladder control”
d. “I have difficulty swallowing food”
a. “I have a hard time with brushing my hair”

A nurse is providing discharge teaching to a client who will be self-administering insulin at home. Which of the following information should the nurse include regarding needle disposal?
a. “secure the cap tightly over the needle before you discard it”
b. “remove the needle from the syringe before you place it in the trash”
c. “you can discard needles in an empty bleach bottle with a lid”
d. “place your storage container in a recycle bin when it is full”
c. “you can discard needles in an empty bleach bottle with a lid”

A nurse is assessing a client who has arteriovenous (AV) graft in the left forearm. Which of the following findings should indicate to the nurse a complication of vascular access?
a. 2+ left radical pulse
b. Absence of a bruit
c. Presence of a palpable thrill
d. Dilated appearance of the AV site
b. Absence of a bruit

A client who is deaf and communicates using sign language is being admitted by a nurse who does not know sign language. Which of the following actions should the nurse take?
a. Familiarize themselves with commonly used sign language
b. Ask a family member to be present during the admission
c. Obtain a board that uses colored pictures as communication
d. Request an interpreter during the initial assessment
d. Request an interpreter during the initial assessment

A nurse is planning care for a client who has unrepaired intertrochanteric fracture and has Buck’s traction placed to the affected leg. Which of the following interventions should the
nurse include?
a. Situate the client’s heel in the heel of the traction boot
b. Apply weights of the traction to total 9.1 kg (20lb)
c. Place the footplate against the foot of the bed
d. Remove the boot for skin inspection every 12 hours.
a. Situate the client’s heel in the heel of the traction boot

A nurse is caring for a client who sustained a spinal cord injury in a diving accident. Which of the following actions should the nurse take?
a. Assess the client’s neurological status every 8 hour
b. Monitor urine output hourly
c. Provide the client with a low-fiber diet
d. Log roll the client every 4 hour
b. Monitor urine output hourly

A nurse is planning care for a client who has a central venous access device for intermittent infusions. Which of the following actions should the nurse include in the plan of care?
a. Flush the catheter using a 10ml syringe
b. Change the dressing every 24 hour
c. Use clean technique when changing the dressing
d. Cleanse the site with povidone-iodine
a. Flush the catheter using a 10ml syringe

A nurse in the emergency department is caring for a client who has a gunshot wound to the abdomen. Which of the following actions should the nurse take first?
a. Check the color of the client’s skin
b. Remove all of the client’s clothing
c. Administer an opioid analgesic
d. Prepare the client for periorbital lavage
a. Check the color of the client’s skin

A nurse is caring for a client following a bronchoscopy. Which of the following actions should the nurse take first?
a. Check the client’s gag reflex
b. Inform the client they might experience a low-grade fever
c. Instruct the client to report bleeding
d. Provide the client with sips of water
a. Check the client’s gag reflex

A nurse is developing a plan of care for a client who is returning from the PACU following a left below-the-knee amputation. Which of the following interventions should the nurse include in the plan?
a. Provide the client with a firm mattress
b. Wrap the client’s residual limb with elastic bandage in a distal to proximal direction
c. Place the client’s residual limb in a dependent position when possible
d. Keep the client in a supine position for 48 hours
a. Provide the client with a firm mattress

A nurse is instructing a client who has a new diagnosis of type 1 diabetes mellitus about the sick-day rules. Which of the following statements by the client indicates an understanding of
the teaching?
a. “I will monitor my blood glucose every 8 hours”
b. “I will consume 250 grams of carbs daily while I’m sick”
c. “I will not take my diabetes medications while I am sick”
d. “I will check urine for ketones if my blood glucose is greater than 240 mg/dl
d. “I will check urine for ketones if my blood glucose is greater than 240 mg/dl

A nurse is reviewing ABG results for a client who has COPD. Which of the following findings should the nurse expect?
a. pH 7.38
b. PaO2 85 mm Hg
c. PaCO2 48 mm Hg
d. HCO3- 25 mEq/l
c. PaCO2 48 mm Hg

A nurse is admitting a client to a medical unit following placement of a permanent pacemaker. Which of the following findings requires further assessment by the nurse?
a. Sneezing
b. Hiccups
c. Presence of a sharp spike prior to the QRS complex on the ECG
d. Presence of intrinsic P waves following a QRS complex on the ECG
b. Hiccups

A nurse is caring for a client who experienced extensive burns to the arms and torso. Which of the following actions should the nurse take regarding the client’s oral nutritional intake?
a. Adhere to scheduled meal times three times daily
b. Encourage the client to eat as many calories as possible
c. Limit the client’s fluid intake to 1,500 ml/day
d. Avoid the use of supplemental feedings throughout the day
b. Encourage the client to eat as many calories as possible

A nurse is planning care for a client who is 1 day postoperative following an open cholecystectomy. Which of the following interventions should the nurse include in the plan of
care?
a. Place pillows under the client’s knees
b. Apply compression stockings to the lower extremities
c. Avoid use of anticoagulants
d. Discourage leg exercises while in bed
b. Apply compression stockings to the lower extremities

A nurse is caring for a client who has duodenal ulcer. Which of the following actions should the nurse take? Exhibit
a. Restrict the client’s fluid intake to 1,000 ml/day
b. Infuse packed RBCs
c. Administer the client’s naproxen prescription
d. Offer a snack before bedtime
b. Infuse packed RBCs

A nurse is assessing an older adult client at a health fair. Which of the following statements by the client is the nurse’s priority?
a. “I can’t seem to get reading materials far enough away to see the words”
b. “I’m having more difficulty telling the difference between blues and greens”
c. “I’ve noticed that there is a gray ring around the colored part of my eye”
d. “In the last day, I have had a severe headache and pain around my right eye”
d. “In the last day, I have had a severe headache and pain around my right eye”

A nurse is caring for an adolescent client who has an acute kidney injury. Which of the following laboratory findings should the nurse anticipate?
a. BUN 8 mg/dl
b. Hgb 20 g/dl
c. Potassium 6.8 mEq/l
d. Creatinine 0.4 mg/dl
c. Potassium 6.8 mEq/l

A nurse is planning care for an older adult client who has Meniere’s disease. Which of the following interventions should the nurse include in the plan?
a. Perform range-of-motion exercises to the client’s neck every 4 hour
b. Limit the client’s fluid intake to 1,500 ml/day
c. Encourage the client to change positions slowly
d. Administer aspirin if the client reports a headache
c. Encourage the client to change positions slowly

A nurse is preparing to receive a client from surgery following a transverse colon resection with colostomy placement. The nurse should expect to assess the stoma at which of the following locations?
a. Upper left abdomen (THIS ONE IS A DIAGRAM CHOOSE THE POINT ON THE TOP LEFT OF
ABDOMEN)

A nurse is admitting a client to the emergency department after a gunshot wound to the abdomen. Which of the following actions should the nurse take to help prevent the onset of acute kidney failure?
a. Initiate beta blocker therapy
b. Insert a urinary catheter
c. Prepare the client for an intravenous pyelogram
d. Administer IV fluids to the client
d. Administer IV fluids to the client

A nurse is preparing to administer 1 unit of packed RBCs to an adult client. Which of the following actions should the nurse plan to take?
a. Administer through a 22-gauge IV catheter
b. Prime the IV tubing with 0.45% sodium chloride
c. Complete the transfusion within 2 hour
d. Slow the transfusion rate if the client reports itching
c. Complete the transfusion within 2 hour

A nurse is planning care for a client who has developed nephrotic syndrome. Which of the following dietary recommendations should the nurse include?
a. Increase phosphorus intake
b. Decrease protein intake
c. Increase potassium intake
d. Decrease carbohydrate intake
b. Decrease protein intake

A nurse is caring for an older adult client who has dementia. Which of the following questions should the nurse ask to assess the client’s abstract thinking?
a. “can you count backwards from 100 intervals of 7?”
b. “what is meant by the saying, don’t beat around the brush?
c. “what do you understand about your condition?”
d. “can you tell me the state where you were born?”
b. “what is meant by the saying, don’t beat around the brush?

A nurse is caring for a client who has cervical cancer and is receiving brachytherapy. Which of the following actions should the nurse take?
a. Keep the soiled bed linens in the client’s room
b. Instruct visitors to remain 3 feet from the client
c. Discard the radioactive device in the client’s trash can
d. Limit time for visitors to 2 hour per day
a. Keep the soiled bed linens in the client’s room

A nurse is preparing a client for a lumbar puncture. Which of the following images indicates the position should the nurse assist the client into for this procedure?
a. Side-lying (THIS IS A DIAGRAM CHOOSE THE PERSON IN A FETAL POSITION)

A nurse is caring for a client who has cervical cancer and a sealed radiation implant. Which of the following actions should the nurse take?
a. Place long-handled forceps at the client’s bedside
b. Attach a dosimeter badge to the client’s gown
c. Leave unused equipment in the client’s room until discharge
d. Move the client’s soiled linens to a designated container outside the room
a. Place long-handled forceps at the client’s bedside

A nurse is teaching a client who has Graves’ disease about recognizing the manifestations of the thyroid storm. Which of the following findings should the nurse include in the teaching?
a. Increased temperature
b. Decreased HR
c. Hypotension
d. Lethargy
a. Increased temperature

A nurse is caring for a client who is postoperative following a complete thyroidectomy. Which of the following findings is the priority for the nurse to report to the provider?
a. Serosanguineous drainage
b. Muscle twitching
c. Client report of nausea
d. Client report incisional pain
b. Muscle twitching

A nurse is reviewing ECG rhythm strips for a group of clients. The nurse should identify that which of the following rhythms indicates bradycardia?
pick the brady strip

A nurse is caring for a client who is receiving epidural analgesics. Which of the following assessment findings in the nurse’s priority?
a. Bladder distention
b. Hypoactive bowel sounds
c. Hypotension
d. Weakness to lower extremities
c. Hypotension

A nurse is planning care for a client who has status epilepticus. Which of the following interventions is the nurse’s priority to include?
a. Turn the client to the lateral position during seizure activity
b. Provide the client oxygen at 6 l/min using a nasal cannula
c. Administer phenytoin IV bolus to the client
d. Administer diazepam intravenously to the client
a. Turn the client to the lateral position during seizure activity

A nurse is caring for a client following a below-the knee amputation. The client states. “my life is over.” Which of the following responses should the nurse make?
a. “you are upset. We can talk about this later?”
b. “would you like to meet with another client who is an amputee?”
c. “why do you think your life is over?”
d. “most people can adjust following this surgery”
b. “would you like to meet with another client who is an amputee?”

A nurse in a clinic is providing preventive teaching to an older adult client during a well visit. The nurse should instruct the client that which of the following immunizations are recommended for healthy adults after the age of 60? Select All That Apply
a. Herpes zoster
b. Influenza
c. Meningococcal
d. Human papillomavirus
e. Pneumococcal polysaccharide
a. Herpes zoster
b. Influenza
e. Pneumococcal polysaccharide

A nurse is planning care for a client who has dementia. Which of the following interventions should the nurse plan to include?
a. Turn off all lights in the client’s room at night
b. Place the client’s bed at the lowest height
c. Request a prescription for a nightly sedative
d. Assist the client with toileting at least once every 4 hour
b. Place the client’s bed at the lowest height

A nurse is assessing a client who has a new diagnosis of type 1 diabetes mellitus. Which of the following findings indicates that the client is experiencing hypoglycemia?
a. Abdominal cramping
b. Increased perspiration
c. Dehydration
d. Fruity odor to breath
b. Increased perspiration

A nurse in the PACU is assessing a client who is postoperative following general anesthesia. Which of the following findings is the priority to address?
a. Vomiting upon arousal
b. Decreased body temperature
c. Indistinct, rambling speech
d. Piloerection of the skin
a. Vomiting upon arousal

A nurse is caring for a client who has hypervolemia. Which of the following is an expected assessment finding?
a. Bradycardia
b. Hypotension
c. Loss of skin turgor
d. Weight gain
d. Weight gain

A nurse is teaching about measures to prevent recurring urinary tract infections with a female client. Which of the following information should the nurse include in the teaching? Select All That Apply
a. Take a warm bubble bath daily
b. Void every 6 hour during the day
c. Drink low-fructose cranberry juice
d. Wipe the perineal area from front to back after urinating
e. Drink 3L of fluids daily
c. Drink low-fructose cranberry juice
d. Wipe the perineal area from front to back after urinating
e. Drink 3L of fluids daily

A nurse is caring for a client following a cardiac catheterization who has hives and urticaria following administration of IV contrast dye. Which of the following medications should the nurse plan to administer?
a. Spironolactone
b. Desmopressin
c. Metoclopramide
d. Diphenhydramine
d. Diphenhydramine

A home care nurse is planning to use nonpharmacological pain relief measures for an older adult client who has severe chronic back pain. Which of the following guidelines should the nurse use?
a. Discontinue opioids before trying nonpharmacological methods of pain relief
b. Use imagery with clients who have difficulty with focus and concentration
c. Distraction changes the client’s perception of pain, but does not affect the cause
d. Pain relief from the use of heat and cold continues for several hours after removal of the
stimulus
c. Distraction changes the client’s perception of pain, but does not affect the cause

A nurse is caring for a female client who is receiving total parental nutrition without fat emulsion. Which of the following findings should the nurse report?
a. Crackles in the bilateral lung bases
b. Weight gain of 1.3 kg (3lb) over the past 7 days
c. Triglyceride 110 mg/dl
d. Bowel sounds absent in lower quadrants
a. Crackles in the bilateral lung bases

Leave a Comment

Scroll to Top